Limit of sequence (by definiton)Prove:$limlimits_nto infty frac(-1)^nn = 0$How to find the limit of this rational sequence?2 Sub-limits of sequence converge epsilon proofLimit of a function involving a sequence.Understanding how to use $epsilon-delta$ definition of a limitFind the limit of a sequence defined recursivelyCubic root of sequence translates into cubic root of limit?Prove that $a_n=fracnn+1$ is convergentProving Limit of sequence using Epsilon-N definitionProof of limit of quotient sequence property

Examples of problems with non-convex constraint functions but convex feasible region

How come Aboriginal Australians didn't manage to raise their civilization levels to that of other continents?

Should I respond to a sabotage accusation e-mail at work?

Why do military jets sometimes have elevators in a depressed position when parked?

Why doesn't English employ an H in front of the name Ares?

What is the white square near the viewfinder of the Fujica GW690?

Green Flame Blade and Negative Modifiers

I don't want my ls command in my script to print results on screen

Is it okay to request a vegetarian only microwave at work ? If, yes, what's the proper way to do it?

How much does freezing grapes longer sweeten them more?

Visualize a large int

Did the US push the Kurds to lower their defences against Turkey in the months preceding the latest Turkish military operation against them?

Prisoner's dilemma formulation for children

Why it is a big deal whether or not Adam Schiff talked to the whistleblower?

When and why did the House rules change to permit an inquiry without a vote?

What would a chair for a Human with a Tail look like?

How can I seal 8 inch round holes in my siding?

How to cut a perfect shape out of 4cm oak?

What are these objects near the Cosmonaut's faces?

Did it take 3 minutes to reload a musket when the second amendment to the US constitution was ratified?

Are there any Baryons that have quark-antiquark combinations?

Confused about the meaning of the word "open" in this sentence

Generate an array with custom index

How would a race of humanoids with tails design [vehicle] seats?



Limit of sequence (by definiton)


Prove:$limlimits_nto infty frac(-1)^nn = 0$How to find the limit of this rational sequence?2 Sub-limits of sequence converge epsilon proofLimit of a function involving a sequence.Understanding how to use $epsilon-delta$ definition of a limitFind the limit of a sequence defined recursivelyCubic root of sequence translates into cubic root of limit?Prove that $a_n=fracnn+1$ is convergentProving Limit of sequence using Epsilon-N definitionProof of limit of quotient sequence property






.everyoneloves__top-leaderboard:empty,.everyoneloves__mid-leaderboard:empty,.everyoneloves__bot-mid-leaderboard:empty
margin-bottom:0;

.everyonelovesstackoverflowposition:absolute;height:1px;width:1px;opacity:0;top:0;left:0;pointer-events:none;








5














$begingroup$


I have this assignment:



$$lim_nto infty frac(-1)^n-3n^2=0$$



I have to prove that limit by definition, but I am stuck with



$$|a_n - 0|< epsilon$$



Dont know how to deal with that absolute value.



The solution is



$$n_0=left[frac2sqrtepsilonright]+1$$










share|cite|improve this question









New contributor



naruto25 is a new contributor to this site. Take care in asking for clarification, commenting, and answering.
Check out our Code of Conduct.







$endgroup$














  • $begingroup$
    Have you tried to see what the numerator is for the first few $n$? That might give you an idea of how to deal with the absolute value. Note that the denominator is always positive.
    $endgroup$
    – Theoretical Economist
    10 hours ago

















5














$begingroup$


I have this assignment:



$$lim_nto infty frac(-1)^n-3n^2=0$$



I have to prove that limit by definition, but I am stuck with



$$|a_n - 0|< epsilon$$



Dont know how to deal with that absolute value.



The solution is



$$n_0=left[frac2sqrtepsilonright]+1$$










share|cite|improve this question









New contributor



naruto25 is a new contributor to this site. Take care in asking for clarification, commenting, and answering.
Check out our Code of Conduct.







$endgroup$














  • $begingroup$
    Have you tried to see what the numerator is for the first few $n$? That might give you an idea of how to deal with the absolute value. Note that the denominator is always positive.
    $endgroup$
    – Theoretical Economist
    10 hours ago













5












5








5





$begingroup$


I have this assignment:



$$lim_nto infty frac(-1)^n-3n^2=0$$



I have to prove that limit by definition, but I am stuck with



$$|a_n - 0|< epsilon$$



Dont know how to deal with that absolute value.



The solution is



$$n_0=left[frac2sqrtepsilonright]+1$$










share|cite|improve this question









New contributor



naruto25 is a new contributor to this site. Take care in asking for clarification, commenting, and answering.
Check out our Code of Conduct.







$endgroup$




I have this assignment:



$$lim_nto infty frac(-1)^n-3n^2=0$$



I have to prove that limit by definition, but I am stuck with



$$|a_n - 0|< epsilon$$



Dont know how to deal with that absolute value.



The solution is



$$n_0=left[frac2sqrtepsilonright]+1$$







calculus limits






share|cite|improve this question









New contributor



naruto25 is a new contributor to this site. Take care in asking for clarification, commenting, and answering.
Check out our Code of Conduct.











share|cite|improve this question









New contributor



naruto25 is a new contributor to this site. Take care in asking for clarification, commenting, and answering.
Check out our Code of Conduct.








share|cite|improve this question




share|cite|improve this question








edited 9 hours ago









Don Thousand

5,0441 gold badge11 silver badges35 bronze badges




5,0441 gold badge11 silver badges35 bronze badges






New contributor



naruto25 is a new contributor to this site. Take care in asking for clarification, commenting, and answering.
Check out our Code of Conduct.








asked 10 hours ago









naruto25naruto25

576 bronze badges




576 bronze badges




New contributor



naruto25 is a new contributor to this site. Take care in asking for clarification, commenting, and answering.
Check out our Code of Conduct.




New contributor




naruto25 is a new contributor to this site. Take care in asking for clarification, commenting, and answering.
Check out our Code of Conduct.
















  • $begingroup$
    Have you tried to see what the numerator is for the first few $n$? That might give you an idea of how to deal with the absolute value. Note that the denominator is always positive.
    $endgroup$
    – Theoretical Economist
    10 hours ago
















  • $begingroup$
    Have you tried to see what the numerator is for the first few $n$? That might give you an idea of how to deal with the absolute value. Note that the denominator is always positive.
    $endgroup$
    – Theoretical Economist
    10 hours ago















$begingroup$
Have you tried to see what the numerator is for the first few $n$? That might give you an idea of how to deal with the absolute value. Note that the denominator is always positive.
$endgroup$
– Theoretical Economist
10 hours ago




$begingroup$
Have you tried to see what the numerator is for the first few $n$? That might give you an idea of how to deal with the absolute value. Note that the denominator is always positive.
$endgroup$
– Theoretical Economist
10 hours ago










3 Answers
3






active

oldest

votes


















3
















$begingroup$

We have that



$$frac-2n^2lefrac(-1)^n-3n^2le frac-4n^2 $$



and then



$$frac2n^2leleft|frac(-1)^n-3n^2right|le frac4n^2 $$



therefore



$$left|frac(-1)^n-3n^2right|le frac4n^2 <epsilon implies n^2 > frac4epsilon implies n>frac2sqrtepsilon$$



therefore it suffices to take



$$n_0=overbracecolorblueleft[frac2sqrtepsilonright]^colorredtextinteger part+1$$






share|cite|improve this answer












$endgroup$










  • 1




    $begingroup$
    Since we need $n>frac2sqrtepsilon$ we take the integer part of that and add +1.
    $endgroup$
    – gimusi
    10 hours ago






  • 1




    $begingroup$
    @naruto25 By definition of limit we are looking for $n_0 in mathbb N$ but $frac2sqrtepsilon in mathbb R$ therefore we need to extract the integer part and then add $1$. Try with a numerica example: suppose we obtain $n> 100.45$ then we take $n_0=100+1$.
    $endgroup$
    – gimusi
    9 hours ago






  • 1




    $begingroup$
    @naruto25 We are taking an upper bound that is $4/n^2$, we don't need the exact bound but a simple upper which works. For example we also could take $6/n^2$ and the proof works at the same way.
    $endgroup$
    – gimusi
    9 hours ago






  • 1




    $begingroup$
    @naruto25 We are thinking $epsilon$ as a small number, for example $epsilon=.002 implies sqrt epsilon =0.04472...$ and $frac2sqrtepsilon>44.72...$ then we can take $n_0=45$.
    $endgroup$
    – gimusi
    9 hours ago






  • 1




    $begingroup$
    @naruto25 Exactly!
    $endgroup$
    – gimusi
    9 hours ago


















4
















$begingroup$

Given $epsilon>0 $, we should find $N$ such that if $nge N$ then $$|frac(-1)^n-3n^2|<epsilon$$



but



$$|frac(-1)^n-3n^2|le |frac(-1)^nn^2|+|frac3n^2|le frac4n^2$$



thus it is sufficient to find one $N$ such that,



$$nge N ;; implies ;; frac4n^2<epsilon$$



or



$$nge N ;; implies ;; n^2 > frac4epsilon$$



or
$$nge N ;; implies ;; n > frac2sqrtepsilon$$



So, each $N$ satisfying $N>frac2sqrtepsilon$ will work.



the smallest of these $N$ is



$$lfloor frac2sqrtepsilon rfloor +1$$






share|cite|improve this answer










$endgroup$














  • $begingroup$
    So if I had epsilon=1000000 and then it was 0. I do not have null member of that sequence and thus I have to add that 1?
    $endgroup$
    – naruto25
    10 hours ago










  • $begingroup$
    If $epsilon=1000000$ then $N=1$
    $endgroup$
    – hamam_Abdallah
    10 hours ago











  • $begingroup$
    In general, $epsilon$ Must be Small. Say if $epsilon =0.000001$ then $N =?$
    $endgroup$
    – hamam_Abdallah
    10 hours ago










  • $begingroup$
    I am still curious why do we have to add that one to that solution. If I had epsilon=1, then the first member of sequence(n0) from which that limit starts should be 2. And every next member should fit in distance epsilon.
    $endgroup$
    – naruto25
    10 hours ago










  • $begingroup$
    @naruto25 If it is true for small epsilons, it will be also true for greater epsilons.
    $endgroup$
    – hamam_Abdallah
    10 hours ago


















1
















$begingroup$

Notice, that:



$$frac(-1)^n-3n^2 geq frac-4n^2$$



And is always negative. So for every $epsilon$ you can find $N$ ($=frac2sqrtepsilon)$, that for every $n>N$ the following holds: $$|frac(-1)^n-3n^2| leq epsilon$$






share|cite|improve this answer










$endgroup$
















    Your Answer








    StackExchange.ready(function()
    var channelOptions =
    tags: "".split(" "),
    id: "69"
    ;
    initTagRenderer("".split(" "), "".split(" "), channelOptions);

    StackExchange.using("externalEditor", function()
    // Have to fire editor after snippets, if snippets enabled
    if (StackExchange.settings.snippets.snippetsEnabled)
    StackExchange.using("snippets", function()
    createEditor();
    );

    else
    createEditor();

    );

    function createEditor()
    StackExchange.prepareEditor(
    heartbeatType: 'answer',
    autoActivateHeartbeat: false,
    convertImagesToLinks: true,
    noModals: true,
    showLowRepImageUploadWarning: true,
    reputationToPostImages: 10,
    bindNavPrevention: true,
    postfix: "",
    imageUploader:
    brandingHtml: "Powered by u003ca class="icon-imgur-white" href="https://imgur.com/"u003eu003c/au003e",
    contentPolicyHtml: "User contributions licensed under u003ca href="https://creativecommons.org/licenses/by-sa/4.0/"u003ecc by-sa 4.0 with attribution requiredu003c/au003e u003ca href="https://stackoverflow.com/legal/content-policy"u003e(content policy)u003c/au003e",
    allowUrls: true
    ,
    noCode: true, onDemand: true,
    discardSelector: ".discard-answer"
    ,immediatelyShowMarkdownHelp:true
    );



    );







    naruto25 is a new contributor. Be nice, and check out our Code of Conduct.









    draft saved

    draft discarded
















    StackExchange.ready(
    function ()
    StackExchange.openid.initPostLogin('.new-post-login', 'https%3a%2f%2fmath.stackexchange.com%2fquestions%2f3389941%2flimit-of-sequence-by-definiton%23new-answer', 'question_page');

    );

    Post as a guest















    Required, but never shown


























    3 Answers
    3






    active

    oldest

    votes








    3 Answers
    3






    active

    oldest

    votes









    active

    oldest

    votes






    active

    oldest

    votes









    3
















    $begingroup$

    We have that



    $$frac-2n^2lefrac(-1)^n-3n^2le frac-4n^2 $$



    and then



    $$frac2n^2leleft|frac(-1)^n-3n^2right|le frac4n^2 $$



    therefore



    $$left|frac(-1)^n-3n^2right|le frac4n^2 <epsilon implies n^2 > frac4epsilon implies n>frac2sqrtepsilon$$



    therefore it suffices to take



    $$n_0=overbracecolorblueleft[frac2sqrtepsilonright]^colorredtextinteger part+1$$






    share|cite|improve this answer












    $endgroup$










    • 1




      $begingroup$
      Since we need $n>frac2sqrtepsilon$ we take the integer part of that and add +1.
      $endgroup$
      – gimusi
      10 hours ago






    • 1




      $begingroup$
      @naruto25 By definition of limit we are looking for $n_0 in mathbb N$ but $frac2sqrtepsilon in mathbb R$ therefore we need to extract the integer part and then add $1$. Try with a numerica example: suppose we obtain $n> 100.45$ then we take $n_0=100+1$.
      $endgroup$
      – gimusi
      9 hours ago






    • 1




      $begingroup$
      @naruto25 We are taking an upper bound that is $4/n^2$, we don't need the exact bound but a simple upper which works. For example we also could take $6/n^2$ and the proof works at the same way.
      $endgroup$
      – gimusi
      9 hours ago






    • 1




      $begingroup$
      @naruto25 We are thinking $epsilon$ as a small number, for example $epsilon=.002 implies sqrt epsilon =0.04472...$ and $frac2sqrtepsilon>44.72...$ then we can take $n_0=45$.
      $endgroup$
      – gimusi
      9 hours ago






    • 1




      $begingroup$
      @naruto25 Exactly!
      $endgroup$
      – gimusi
      9 hours ago















    3
















    $begingroup$

    We have that



    $$frac-2n^2lefrac(-1)^n-3n^2le frac-4n^2 $$



    and then



    $$frac2n^2leleft|frac(-1)^n-3n^2right|le frac4n^2 $$



    therefore



    $$left|frac(-1)^n-3n^2right|le frac4n^2 <epsilon implies n^2 > frac4epsilon implies n>frac2sqrtepsilon$$



    therefore it suffices to take



    $$n_0=overbracecolorblueleft[frac2sqrtepsilonright]^colorredtextinteger part+1$$






    share|cite|improve this answer












    $endgroup$










    • 1




      $begingroup$
      Since we need $n>frac2sqrtepsilon$ we take the integer part of that and add +1.
      $endgroup$
      – gimusi
      10 hours ago






    • 1




      $begingroup$
      @naruto25 By definition of limit we are looking for $n_0 in mathbb N$ but $frac2sqrtepsilon in mathbb R$ therefore we need to extract the integer part and then add $1$. Try with a numerica example: suppose we obtain $n> 100.45$ then we take $n_0=100+1$.
      $endgroup$
      – gimusi
      9 hours ago






    • 1




      $begingroup$
      @naruto25 We are taking an upper bound that is $4/n^2$, we don't need the exact bound but a simple upper which works. For example we also could take $6/n^2$ and the proof works at the same way.
      $endgroup$
      – gimusi
      9 hours ago






    • 1




      $begingroup$
      @naruto25 We are thinking $epsilon$ as a small number, for example $epsilon=.002 implies sqrt epsilon =0.04472...$ and $frac2sqrtepsilon>44.72...$ then we can take $n_0=45$.
      $endgroup$
      – gimusi
      9 hours ago






    • 1




      $begingroup$
      @naruto25 Exactly!
      $endgroup$
      – gimusi
      9 hours ago













    3














    3










    3







    $begingroup$

    We have that



    $$frac-2n^2lefrac(-1)^n-3n^2le frac-4n^2 $$



    and then



    $$frac2n^2leleft|frac(-1)^n-3n^2right|le frac4n^2 $$



    therefore



    $$left|frac(-1)^n-3n^2right|le frac4n^2 <epsilon implies n^2 > frac4epsilon implies n>frac2sqrtepsilon$$



    therefore it suffices to take



    $$n_0=overbracecolorblueleft[frac2sqrtepsilonright]^colorredtextinteger part+1$$






    share|cite|improve this answer












    $endgroup$



    We have that



    $$frac-2n^2lefrac(-1)^n-3n^2le frac-4n^2 $$



    and then



    $$frac2n^2leleft|frac(-1)^n-3n^2right|le frac4n^2 $$



    therefore



    $$left|frac(-1)^n-3n^2right|le frac4n^2 <epsilon implies n^2 > frac4epsilon implies n>frac2sqrtepsilon$$



    therefore it suffices to take



    $$n_0=overbracecolorblueleft[frac2sqrtepsilonright]^colorredtextinteger part+1$$







    share|cite|improve this answer















    share|cite|improve this answer




    share|cite|improve this answer








    edited 10 hours ago

























    answered 10 hours ago









    gimusigimusi

    97.4k9 gold badges47 silver badges96 bronze badges




    97.4k9 gold badges47 silver badges96 bronze badges










    • 1




      $begingroup$
      Since we need $n>frac2sqrtepsilon$ we take the integer part of that and add +1.
      $endgroup$
      – gimusi
      10 hours ago






    • 1




      $begingroup$
      @naruto25 By definition of limit we are looking for $n_0 in mathbb N$ but $frac2sqrtepsilon in mathbb R$ therefore we need to extract the integer part and then add $1$. Try with a numerica example: suppose we obtain $n> 100.45$ then we take $n_0=100+1$.
      $endgroup$
      – gimusi
      9 hours ago






    • 1




      $begingroup$
      @naruto25 We are taking an upper bound that is $4/n^2$, we don't need the exact bound but a simple upper which works. For example we also could take $6/n^2$ and the proof works at the same way.
      $endgroup$
      – gimusi
      9 hours ago






    • 1




      $begingroup$
      @naruto25 We are thinking $epsilon$ as a small number, for example $epsilon=.002 implies sqrt epsilon =0.04472...$ and $frac2sqrtepsilon>44.72...$ then we can take $n_0=45$.
      $endgroup$
      – gimusi
      9 hours ago






    • 1




      $begingroup$
      @naruto25 Exactly!
      $endgroup$
      – gimusi
      9 hours ago












    • 1




      $begingroup$
      Since we need $n>frac2sqrtepsilon$ we take the integer part of that and add +1.
      $endgroup$
      – gimusi
      10 hours ago






    • 1




      $begingroup$
      @naruto25 By definition of limit we are looking for $n_0 in mathbb N$ but $frac2sqrtepsilon in mathbb R$ therefore we need to extract the integer part and then add $1$. Try with a numerica example: suppose we obtain $n> 100.45$ then we take $n_0=100+1$.
      $endgroup$
      – gimusi
      9 hours ago






    • 1




      $begingroup$
      @naruto25 We are taking an upper bound that is $4/n^2$, we don't need the exact bound but a simple upper which works. For example we also could take $6/n^2$ and the proof works at the same way.
      $endgroup$
      – gimusi
      9 hours ago






    • 1




      $begingroup$
      @naruto25 We are thinking $epsilon$ as a small number, for example $epsilon=.002 implies sqrt epsilon =0.04472...$ and $frac2sqrtepsilon>44.72...$ then we can take $n_0=45$.
      $endgroup$
      – gimusi
      9 hours ago






    • 1




      $begingroup$
      @naruto25 Exactly!
      $endgroup$
      – gimusi
      9 hours ago







    1




    1




    $begingroup$
    Since we need $n>frac2sqrtepsilon$ we take the integer part of that and add +1.
    $endgroup$
    – gimusi
    10 hours ago




    $begingroup$
    Since we need $n>frac2sqrtepsilon$ we take the integer part of that and add +1.
    $endgroup$
    – gimusi
    10 hours ago




    1




    1




    $begingroup$
    @naruto25 By definition of limit we are looking for $n_0 in mathbb N$ but $frac2sqrtepsilon in mathbb R$ therefore we need to extract the integer part and then add $1$. Try with a numerica example: suppose we obtain $n> 100.45$ then we take $n_0=100+1$.
    $endgroup$
    – gimusi
    9 hours ago




    $begingroup$
    @naruto25 By definition of limit we are looking for $n_0 in mathbb N$ but $frac2sqrtepsilon in mathbb R$ therefore we need to extract the integer part and then add $1$. Try with a numerica example: suppose we obtain $n> 100.45$ then we take $n_0=100+1$.
    $endgroup$
    – gimusi
    9 hours ago




    1




    1




    $begingroup$
    @naruto25 We are taking an upper bound that is $4/n^2$, we don't need the exact bound but a simple upper which works. For example we also could take $6/n^2$ and the proof works at the same way.
    $endgroup$
    – gimusi
    9 hours ago




    $begingroup$
    @naruto25 We are taking an upper bound that is $4/n^2$, we don't need the exact bound but a simple upper which works. For example we also could take $6/n^2$ and the proof works at the same way.
    $endgroup$
    – gimusi
    9 hours ago




    1




    1




    $begingroup$
    @naruto25 We are thinking $epsilon$ as a small number, for example $epsilon=.002 implies sqrt epsilon =0.04472...$ and $frac2sqrtepsilon>44.72...$ then we can take $n_0=45$.
    $endgroup$
    – gimusi
    9 hours ago




    $begingroup$
    @naruto25 We are thinking $epsilon$ as a small number, for example $epsilon=.002 implies sqrt epsilon =0.04472...$ and $frac2sqrtepsilon>44.72...$ then we can take $n_0=45$.
    $endgroup$
    – gimusi
    9 hours ago




    1




    1




    $begingroup$
    @naruto25 Exactly!
    $endgroup$
    – gimusi
    9 hours ago




    $begingroup$
    @naruto25 Exactly!
    $endgroup$
    – gimusi
    9 hours ago













    4
















    $begingroup$

    Given $epsilon>0 $, we should find $N$ such that if $nge N$ then $$|frac(-1)^n-3n^2|<epsilon$$



    but



    $$|frac(-1)^n-3n^2|le |frac(-1)^nn^2|+|frac3n^2|le frac4n^2$$



    thus it is sufficient to find one $N$ such that,



    $$nge N ;; implies ;; frac4n^2<epsilon$$



    or



    $$nge N ;; implies ;; n^2 > frac4epsilon$$



    or
    $$nge N ;; implies ;; n > frac2sqrtepsilon$$



    So, each $N$ satisfying $N>frac2sqrtepsilon$ will work.



    the smallest of these $N$ is



    $$lfloor frac2sqrtepsilon rfloor +1$$






    share|cite|improve this answer










    $endgroup$














    • $begingroup$
      So if I had epsilon=1000000 and then it was 0. I do not have null member of that sequence and thus I have to add that 1?
      $endgroup$
      – naruto25
      10 hours ago










    • $begingroup$
      If $epsilon=1000000$ then $N=1$
      $endgroup$
      – hamam_Abdallah
      10 hours ago











    • $begingroup$
      In general, $epsilon$ Must be Small. Say if $epsilon =0.000001$ then $N =?$
      $endgroup$
      – hamam_Abdallah
      10 hours ago










    • $begingroup$
      I am still curious why do we have to add that one to that solution. If I had epsilon=1, then the first member of sequence(n0) from which that limit starts should be 2. And every next member should fit in distance epsilon.
      $endgroup$
      – naruto25
      10 hours ago










    • $begingroup$
      @naruto25 If it is true for small epsilons, it will be also true for greater epsilons.
      $endgroup$
      – hamam_Abdallah
      10 hours ago















    4
















    $begingroup$

    Given $epsilon>0 $, we should find $N$ such that if $nge N$ then $$|frac(-1)^n-3n^2|<epsilon$$



    but



    $$|frac(-1)^n-3n^2|le |frac(-1)^nn^2|+|frac3n^2|le frac4n^2$$



    thus it is sufficient to find one $N$ such that,



    $$nge N ;; implies ;; frac4n^2<epsilon$$



    or



    $$nge N ;; implies ;; n^2 > frac4epsilon$$



    or
    $$nge N ;; implies ;; n > frac2sqrtepsilon$$



    So, each $N$ satisfying $N>frac2sqrtepsilon$ will work.



    the smallest of these $N$ is



    $$lfloor frac2sqrtepsilon rfloor +1$$






    share|cite|improve this answer










    $endgroup$














    • $begingroup$
      So if I had epsilon=1000000 and then it was 0. I do not have null member of that sequence and thus I have to add that 1?
      $endgroup$
      – naruto25
      10 hours ago










    • $begingroup$
      If $epsilon=1000000$ then $N=1$
      $endgroup$
      – hamam_Abdallah
      10 hours ago











    • $begingroup$
      In general, $epsilon$ Must be Small. Say if $epsilon =0.000001$ then $N =?$
      $endgroup$
      – hamam_Abdallah
      10 hours ago










    • $begingroup$
      I am still curious why do we have to add that one to that solution. If I had epsilon=1, then the first member of sequence(n0) from which that limit starts should be 2. And every next member should fit in distance epsilon.
      $endgroup$
      – naruto25
      10 hours ago










    • $begingroup$
      @naruto25 If it is true for small epsilons, it will be also true for greater epsilons.
      $endgroup$
      – hamam_Abdallah
      10 hours ago













    4














    4










    4







    $begingroup$

    Given $epsilon>0 $, we should find $N$ such that if $nge N$ then $$|frac(-1)^n-3n^2|<epsilon$$



    but



    $$|frac(-1)^n-3n^2|le |frac(-1)^nn^2|+|frac3n^2|le frac4n^2$$



    thus it is sufficient to find one $N$ such that,



    $$nge N ;; implies ;; frac4n^2<epsilon$$



    or



    $$nge N ;; implies ;; n^2 > frac4epsilon$$



    or
    $$nge N ;; implies ;; n > frac2sqrtepsilon$$



    So, each $N$ satisfying $N>frac2sqrtepsilon$ will work.



    the smallest of these $N$ is



    $$lfloor frac2sqrtepsilon rfloor +1$$






    share|cite|improve this answer










    $endgroup$



    Given $epsilon>0 $, we should find $N$ such that if $nge N$ then $$|frac(-1)^n-3n^2|<epsilon$$



    but



    $$|frac(-1)^n-3n^2|le |frac(-1)^nn^2|+|frac3n^2|le frac4n^2$$



    thus it is sufficient to find one $N$ such that,



    $$nge N ;; implies ;; frac4n^2<epsilon$$



    or



    $$nge N ;; implies ;; n^2 > frac4epsilon$$



    or
    $$nge N ;; implies ;; n > frac2sqrtepsilon$$



    So, each $N$ satisfying $N>frac2sqrtepsilon$ will work.



    the smallest of these $N$ is



    $$lfloor frac2sqrtepsilon rfloor +1$$







    share|cite|improve this answer













    share|cite|improve this answer




    share|cite|improve this answer










    answered 10 hours ago









    hamam_Abdallahhamam_Abdallah

    40k2 gold badges17 silver badges35 bronze badges




    40k2 gold badges17 silver badges35 bronze badges














    • $begingroup$
      So if I had epsilon=1000000 and then it was 0. I do not have null member of that sequence and thus I have to add that 1?
      $endgroup$
      – naruto25
      10 hours ago










    • $begingroup$
      If $epsilon=1000000$ then $N=1$
      $endgroup$
      – hamam_Abdallah
      10 hours ago











    • $begingroup$
      In general, $epsilon$ Must be Small. Say if $epsilon =0.000001$ then $N =?$
      $endgroup$
      – hamam_Abdallah
      10 hours ago










    • $begingroup$
      I am still curious why do we have to add that one to that solution. If I had epsilon=1, then the first member of sequence(n0) from which that limit starts should be 2. And every next member should fit in distance epsilon.
      $endgroup$
      – naruto25
      10 hours ago










    • $begingroup$
      @naruto25 If it is true for small epsilons, it will be also true for greater epsilons.
      $endgroup$
      – hamam_Abdallah
      10 hours ago
















    • $begingroup$
      So if I had epsilon=1000000 and then it was 0. I do not have null member of that sequence and thus I have to add that 1?
      $endgroup$
      – naruto25
      10 hours ago










    • $begingroup$
      If $epsilon=1000000$ then $N=1$
      $endgroup$
      – hamam_Abdallah
      10 hours ago











    • $begingroup$
      In general, $epsilon$ Must be Small. Say if $epsilon =0.000001$ then $N =?$
      $endgroup$
      – hamam_Abdallah
      10 hours ago










    • $begingroup$
      I am still curious why do we have to add that one to that solution. If I had epsilon=1, then the first member of sequence(n0) from which that limit starts should be 2. And every next member should fit in distance epsilon.
      $endgroup$
      – naruto25
      10 hours ago










    • $begingroup$
      @naruto25 If it is true for small epsilons, it will be also true for greater epsilons.
      $endgroup$
      – hamam_Abdallah
      10 hours ago















    $begingroup$
    So if I had epsilon=1000000 and then it was 0. I do not have null member of that sequence and thus I have to add that 1?
    $endgroup$
    – naruto25
    10 hours ago




    $begingroup$
    So if I had epsilon=1000000 and then it was 0. I do not have null member of that sequence and thus I have to add that 1?
    $endgroup$
    – naruto25
    10 hours ago












    $begingroup$
    If $epsilon=1000000$ then $N=1$
    $endgroup$
    – hamam_Abdallah
    10 hours ago





    $begingroup$
    If $epsilon=1000000$ then $N=1$
    $endgroup$
    – hamam_Abdallah
    10 hours ago













    $begingroup$
    In general, $epsilon$ Must be Small. Say if $epsilon =0.000001$ then $N =?$
    $endgroup$
    – hamam_Abdallah
    10 hours ago




    $begingroup$
    In general, $epsilon$ Must be Small. Say if $epsilon =0.000001$ then $N =?$
    $endgroup$
    – hamam_Abdallah
    10 hours ago












    $begingroup$
    I am still curious why do we have to add that one to that solution. If I had epsilon=1, then the first member of sequence(n0) from which that limit starts should be 2. And every next member should fit in distance epsilon.
    $endgroup$
    – naruto25
    10 hours ago




    $begingroup$
    I am still curious why do we have to add that one to that solution. If I had epsilon=1, then the first member of sequence(n0) from which that limit starts should be 2. And every next member should fit in distance epsilon.
    $endgroup$
    – naruto25
    10 hours ago












    $begingroup$
    @naruto25 If it is true for small epsilons, it will be also true for greater epsilons.
    $endgroup$
    – hamam_Abdallah
    10 hours ago




    $begingroup$
    @naruto25 If it is true for small epsilons, it will be also true for greater epsilons.
    $endgroup$
    – hamam_Abdallah
    10 hours ago











    1
















    $begingroup$

    Notice, that:



    $$frac(-1)^n-3n^2 geq frac-4n^2$$



    And is always negative. So for every $epsilon$ you can find $N$ ($=frac2sqrtepsilon)$, that for every $n>N$ the following holds: $$|frac(-1)^n-3n^2| leq epsilon$$






    share|cite|improve this answer










    $endgroup$



















      1
















      $begingroup$

      Notice, that:



      $$frac(-1)^n-3n^2 geq frac-4n^2$$



      And is always negative. So for every $epsilon$ you can find $N$ ($=frac2sqrtepsilon)$, that for every $n>N$ the following holds: $$|frac(-1)^n-3n^2| leq epsilon$$






      share|cite|improve this answer










      $endgroup$

















        1














        1










        1







        $begingroup$

        Notice, that:



        $$frac(-1)^n-3n^2 geq frac-4n^2$$



        And is always negative. So for every $epsilon$ you can find $N$ ($=frac2sqrtepsilon)$, that for every $n>N$ the following holds: $$|frac(-1)^n-3n^2| leq epsilon$$






        share|cite|improve this answer










        $endgroup$



        Notice, that:



        $$frac(-1)^n-3n^2 geq frac-4n^2$$



        And is always negative. So for every $epsilon$ you can find $N$ ($=frac2sqrtepsilon)$, that for every $n>N$ the following holds: $$|frac(-1)^n-3n^2| leq epsilon$$







        share|cite|improve this answer













        share|cite|improve this answer




        share|cite|improve this answer










        answered 10 hours ago









        AndronicusAndronicus

        7981 gold badge2 silver badges14 bronze badges




        7981 gold badge2 silver badges14 bronze badges
























            naruto25 is a new contributor. Be nice, and check out our Code of Conduct.









            draft saved

            draft discarded

















            naruto25 is a new contributor. Be nice, and check out our Code of Conduct.












            naruto25 is a new contributor. Be nice, and check out our Code of Conduct.











            naruto25 is a new contributor. Be nice, and check out our Code of Conduct.














            Thanks for contributing an answer to Mathematics Stack Exchange!


            • Please be sure to answer the question. Provide details and share your research!

            But avoid


            • Asking for help, clarification, or responding to other answers.

            • Making statements based on opinion; back them up with references or personal experience.

            Use MathJax to format equations. MathJax reference.


            To learn more, see our tips on writing great answers.




            draft saved


            draft discarded














            StackExchange.ready(
            function ()
            StackExchange.openid.initPostLogin('.new-post-login', 'https%3a%2f%2fmath.stackexchange.com%2fquestions%2f3389941%2flimit-of-sequence-by-definiton%23new-answer', 'question_page');

            );

            Post as a guest















            Required, but never shown





















































            Required, but never shown














            Required, but never shown












            Required, but never shown







            Required, but never shown

































            Required, but never shown














            Required, but never shown












            Required, but never shown







            Required, but never shown









            Popular posts from this blog

            Invision Community Contents History See also References External links Navigation menuProprietaryinvisioncommunity.comIPS Community ForumsIPS Community Forumsthis blog entry"License Changes, IP.Board 3.4, and the Future""Interview -- Matt Mecham of Ibforums""CEO Invision Power Board, Matt Mecham Is a Liar, Thief!"IPB License Explanation 1.3, 1.3.1, 2.0, and 2.1ArchivedSecurity Fixes, Updates And Enhancements For IPB 1.3.1Archived"New Demo Accounts - Invision Power Services"the original"New Default Skin"the original"Invision Power Board 3.0.0 and Applications Released"the original"Archived copy"the original"Perpetual licenses being done away with""Release Notes - Invision Power Services""Introducing: IPS Community Suite 4!"Invision Community Release Notes

            Canceling a color specificationRandomly assigning color to Graphics3D objects?Default color for Filling in Mathematica 9Coloring specific elements of sets with a prime modified order in an array plotHow to pick a color differing significantly from the colors already in a given color list?Detection of the text colorColor numbers based on their valueCan color schemes for use with ColorData include opacity specification?My dynamic color schemes

            Tom Holland Mục lục Đầu đời và giáo dục | Sự nghiệp | Cuộc sống cá nhân | Phim tham gia | Giải thưởng và đề cử | Chú thích | Liên kết ngoài | Trình đơn chuyển hướngProfile“Person Details for Thomas Stanley Holland, "England and Wales Birth Registration Index, 1837-2008" — FamilySearch.org”"Meet Tom Holland... the 16-year-old star of The Impossible""Schoolboy actor Tom Holland finds himself in Oscar contention for role in tsunami drama"“Naomi Watts on the Prince William and Harry's reaction to her film about the late Princess Diana”lưu trữ"Holland and Pflueger Are West End's Two New 'Billy Elliots'""I'm so envious of my son, the movie star! British writer Dominic Holland's spent 20 years trying to crack Hollywood - but he's been beaten to it by a very unlikely rival"“Richard and Margaret Povey of Jersey, Channel Islands, UK: Information about Thomas Stanley Holland”"Tom Holland to play Billy Elliot""New Billy Elliot leaving the garage"Billy Elliot the Musical - Tom Holland - Billy"A Tale of four Billys: Tom Holland""The Feel Good Factor""Thames Christian College schoolboys join Myleene Klass for The Feelgood Factor""Government launches £600,000 arts bursaries pilot""BILLY's Chapman, Holland, Gardner & Jackson-Keen Visit Prime Minister""Elton John 'blown away' by Billy Elliot fifth birthday" (video with John's interview and fragments of Holland's performance)"First News interviews Arrietty's Tom Holland"“33rd Critics' Circle Film Awards winners”“National Board of Review Current Awards”Bản gốc"Ron Howard Whaling Tale 'In The Heart Of The Sea' Casts Tom Holland"“'Spider-Man' Finds Tom Holland to Star as New Web-Slinger”lưu trữ“Captain America: Civil War (2016)”“Film Review: ‘Captain America: Civil War’”lưu trữ“‘Captain America: Civil War’ review: Choose your own avenger”lưu trữ“The Lost City of Z reviews”“Sony Pictures and Marvel Studios Find Their 'Spider-Man' Star and Director”“‘Mary Magdalene’, ‘Current War’ & ‘Wind River’ Get 2017 Release Dates From Weinstein”“Lionsgate Unleashing Daisy Ridley & Tom Holland Starrer ‘Chaos Walking’ In Cannes”“PTA's 'Master' Leads Chicago Film Critics Nominations, UPDATED: Houston and Indiana Critics Nominations”“Nominaciones Goya 2013 Telecinco Cinema – ENG”“Jameson Empire Film Awards: Martin Freeman wins best actor for performance in The Hobbit”“34th Annual Young Artist Awards”Bản gốc“Teen Choice Awards 2016—Captain America: Civil War Leads Second Wave of Nominations”“BAFTA Film Award Nominations: ‘La La Land’ Leads Race”“Saturn Awards Nominations 2017: 'Rogue One,' 'Walking Dead' Lead”Tom HollandTom HollandTom HollandTom Hollandmedia.gettyimages.comWorldCat Identities300279794no20130442900000 0004 0355 42791085670554170004732cb16706349t(data)XX5557367